Mathcenter Forum

Mathcenter Forum (https://www.mathcenter.net/forum/index.php)
-   อสมการ (https://www.mathcenter.net/forum/forumdisplay.php?f=18)
-   -   โจทย์หัดเเต่งเองครับ (https://www.mathcenter.net/forum/showthread.php?t=15012)

จูกัดเหลียง 09 พฤศจิกายน 2011 18:35

โจทย์หัดเเต่งเองครับ
 
Let $x,y,z>0$ and $xyz=1$
Prove that $$\frac{1}{x(y^2+z^2)+3}+\frac{1}{y(z^2+x^2)+3}+\frac{1}{z(x^2+y^2)+3}\le \frac{3}{5}$$

Amankris 09 พฤศจิกายน 2011 18:44

อ้างอิง:

ข้อความเดิมเขียนโดยคุณ จูกัดเหลียง (ข้อความที่ 127024)
Let $x,y,z>0$ and $xyz=1$

งดงามมากเลยครับ

BLACK-Dragon 10 พฤศจิกายน 2011 11:35

อ้างอิง:

ข้อความเดิมเขียนโดยคุณ จูกัดเหลียง (ข้อความที่ 127024)
Let $x,y,z>0$ and $xyz=1$
Prove that $$\dfrac{1}{x(y^2+z^2)+3}+\dfrac{1}{y(z^2+x^2)+3}+\dfrac{1}{z(x^2+y^2)+3}\le \dfrac{3}{5}$$

คงอยากให้ใช้ A.M.-G.M. กับส่วนล่างใช่ไหมครับ :)

$a^2+b^2 \ge 2ab \Rightarrow \dfrac{1}{c(a^2+b^2)+3} \le \dfrac{1}{2abc+3}$

$\dfrac{1}{x(y^2+z^2)+3}+\dfrac{1}{y(z^2+x^2)+3}+\dfrac{1}{z(x^2+y^2)+3} \le \dfrac{3}{2xyz+3} =\dfrac{3}{5}$

ไม่รู้ถูกเปล่านะครับ มีโจทย์ (ดัดแปลงมา)



$( a,b,c >0 ,\dfrac{3x}{x+1}+\dfrac{4y}{y+1}+\dfrac{5z}{z+1} =1)$

$$x^3y^4z^5 \le \dfrac{1}{11^{12}}$$

จูกัดเหลียง 10 พฤศจิกายน 2011 11:41

อ้างอิง:

ข้อความเดิมเขียนโดยคุณ BLACK-Dragon (ข้อความที่ 127040)
คงอยากให้ใช้ A.M.-G.M. กับส่วนล่างใช่ไหมครับ :)

ได้เเล้วเหรอครับ ง่ายไปจิงๆเเหะ 555

จูกัดเหลียง 10 พฤศจิกายน 2011 13:20

Let $x,y,z>0$ such that $xy+yz+zx=xyz$
Prove $$\frac{x+2y-z}{z+x}+\frac{y+2z-x}{x+y}+\frac{z+2x-y}{y+z}\ge \frac{xyz}{x+y+z}$$

Thgx0312555 10 พฤศจิกายน 2011 17:03

อ้างอิง:

ข้อความเดิมเขียนโดยคุณ BLACK-Dragon (ข้อความที่ 127040)
คงอยากให้ใช้ A.M.-G.M. กับส่วนล่างใช่ไหมครับ :)

$( x,y,z >0 ,\dfrac{3x}{x+1}+\dfrac{4y}{y+1}+\dfrac{5z}{z+1} =1)$

$$x^3y^4z^5 \le \dfrac{1}{11^{12}}$$



$\dfrac{3x+3-3}{x+1}+\dfrac{4y+4-4}{y+1}+\dfrac{5z+5-5}{z+1} =1$

$\dfrac{-3}{x+1}+\dfrac{-4}{y+1}+\dfrac{-5}{z+1} =-11$
$\dfrac{3}{x+1}+\dfrac{4}{y+1}+\dfrac{5}{z+1} =11$

$\dfrac{12}{\frac{3}{x+1}+\frac{4}{y+1}+\frac{5}{z+1}} =\dfrac{12}{11} $ ---(1)

แต่ $\dfrac{3}{x+1}+\dfrac{4}{y+1}+\dfrac{5}{z+1} \leqslant \dfrac{3}{2\sqrt{x}}+\dfrac{4}{2\sqrt{y}}+\dfrac{5}{2\sqrt{z}}$

(1) จัดรูป;

$\dfrac{12}{\frac{1}{\sqrt{x}}+\frac{1}{\sqrt{x}}+\frac{1}{\sqrt{x}}+...+\frac{1}{\sqrt{z}}} \geqslant \dfrac{6}{11} $

GM-HM;

$\sqrt[24]{x^3y^4z^5} \geqslant \dfrac{6}{11} $

$x^3y^4z^5\geqslant (\frac{6}{11} )^{24} = (\frac{36}{121} )^{12} > \frac{1}{11^{12}} $

$\therefore x^3y^4z^5 \le \dfrac{1}{11^{12}}$ เป็นเท็จ

AnDroMeDa 10 พฤศจิกายน 2011 20:30

อ้างอิง:

ข้อความเดิมเขียนโดยคุณ จูกัดเหลียง (ข้อความที่ 127044)
Let $x,y,z>0$ such that $xy+yz+zx=xyz$
Prove $$\frac{x+2y-z}{z+x}+\frac{y+2z-x}{x+y}+\frac{z+2x-y}{y+z}\ge \frac{xyz}{x+y+z}$$

โดยอสมการโคชี $(\sum_{cyc}^{} \frac{x+2y-z}{z+x})(\sum_{cyc}^{} (x+2y-z)(z+x) )\geqslant (2x+2y+2z)^2 $

แต่ $\sum_{cyc}^{} (x+2y-z)(z+x)=(4xy+4yz+4xz)$
จะแสดงว่า $\frac{(2x+2y+2z)^2}{\sum_{cyc}^{} (x+2y-z)(z+x)} =\frac{(x+y+z)^2}{xy+yz+xz} \geqslant \frac{xyz}{x+y+z}$

จาก$xyz=xy+yz+xz$ได้อสมการสมมูลกับ$ (x+y+z)^3\geqslant (xy+yz+xz)^2\geqslant$
$3xyz(x+y+z)=3(xy+yz+xz)(x+y+z)$ ซึ่งได้ $(x+y+z)^2\geqslant 3(xy+yz+xz)$ เป็นจริงโดยอสมการโคชี

note: ผมทำผิดนะ(บรรทัดแรกสุดนะ)เพราะว่า ถ้าดูจากอสมการโคชีที่ http://www.mathcenter.net/forum/showthread.php?t=2439 แล้วต้องได้ว่า$ \sqrt{\frac{x+2y-z}{x+z} } ,\sqrt{\frac{y+2z-x}{y+x} } ,\sqrt{\frac{z+2x-y}{y+z} } ,\sqrt{\frac{x+2y-z}{z+x} } ,\sqrt{\frac{y+2z-x}{x+y} } ,\sqrt{\frac{z+2x-y}{y+z} } \in \mathbb{R} $ ซึ่งถ้าในรูทติดลบมันก้อจะไม่อยู่ใน$\mathbb{R}$ ตามที่คุณ LightLucifer บอกครับ

LightLucifer 11 พฤศจิกายน 2011 00:33

#7
ผมไม่ได้ทำอสมการมานาน แต่คิดว่าต้อง $(x+2y-z) \geq 0$ ด้วยถึงจะจริงอ่ะครับ
แต่ถ้าผิดก็ขออภัยด้วย ไม่ได้แตะนานละ

ปล. เดี๋ยวนี้ เทพ อสมการเพียบเลยเนอะ บอร์ดนี้ ^^

LightLucifer 11 พฤศจิกายน 2011 00:54

ผมไม่คิดว่าข้อนี้จริงนะครับ

http://www.wolframalpha.com/input/?i=%5Cfrac{x%2B2y-z}{z%2Bx}%2B%5Cfrac{y%2B2z-x}{x%2By}%2B%5Cfrac{z%2B2x-y}{y%2Bz}-%5Cfrac{xyz}{x%2By%2Bz}%2Cx%3D%280.1%2B0.1%2B1000%29%2F1000%2Cy%3Dz%3D%281000%2B0.1%2B0.1%29%2F0.1

Amankris 11 พฤศจิกายน 2011 01:31

#6
พิสูจน์กลับข้าง???

AnDroMeDa 11 พฤศจิกายน 2011 03:45

อ้างอิง:

ข้อความเดิมเขียนโดยคุณ BLACK-Dragon (ข้อความที่ 127040)
ไม่รู้ถูกเปล่านะครับ มีโจทย์ (ดัดแปลงมา)



$( a,b,c >0 ,\dfrac{3x}{x+1}+\dfrac{4y}{y+1}+\dfrac{5z}{z+1} =1)$

$$x^3y^4z^5 \le \dfrac{1}{11^{12}}$$

เลือก $\frac{3x}{x+1}=\frac{5}{24} $,$\frac{4y}{y+1}=\frac{9}{24} $,$\frac{5z}{z+1}=\frac{1}{2} $ ได้ $x=\frac{5}{67}$,$y=\frac{9}{85}$,$z=\frac{1}{9}$
ลองดู http://www.wolframalpha.com/input/?i=%285%2F67%29^3%289%2F85%29^4%281%2F9%29^5-%281%2F11%29^12
ได้ $L.H.S-R.H.S.>0$ ซึ่งไม่จริง

Amankris 11 พฤศจิกายน 2011 04:23

#11
ยกตัวอย่างค้านผิดนะครับ

จูกัดเหลียง 11 พฤศจิกายน 2011 13:46

#9 มั้นต้องสอดคล้องเงื่นไขนะครับ -*-

LightLucifer 11 พฤศจิกายน 2011 14:21

ก็ตรงนิครับ

http://www.wolframalpha.com/input/?i...2B0.1%29%2F0.1

จูกัดเหลียง 11 พฤศจิกายน 2011 14:25

#14 เเล้วทำไมผมพิสูจน์ได้อ่ะครับ -*-

BLACK-Dragon 11 พฤศจิกายน 2011 14:26

อ้างอิง:

ข้อความเดิมเขียนโดยคุณ LightLucifer (ข้อความที่ 127079)

คุณ LightLucifer มาแล้วก็น่าจะมันส์น่าดูเลยครับ

ส่วนข้อของผมนะครับ


LightLucifer 11 พฤศจิกายน 2011 14:52

#15
งั้นลองแสดงวิธีให้ดูหน่อยครับ

Thgx0312555 11 พฤศจิกายน 2011 17:22

#10 พิสูจน์ว่าประพจน์นี้เป็นเท็จครับ

"$x^3y^4z^5 \le \dfrac{1}{11^{12}}$"

BLACK-Dragon 11 พฤศจิกายน 2011 18:05

อ้างอิง:

ข้อความเดิมเขียนโดยคุณ Thgx0312555 (ข้อความที่ 127085)
#10 พิสูจน์ว่าประพจน์นี้เป็นเท็จครับ

"$x^3y^4z^5 \le \dfrac{1}{11^{12}}$"

ทำไมหรอครับ ลองค่อยๆดูครับ

Thgx0312555 12 พฤศจิกายน 2011 07:42

<<อันนี้ผิด>>
$\dfrac{3x+3-3}{x+1}+\dfrac{4y+4-4}{y+1}+\dfrac{5z+5-5}{z+1} =1$

$\dfrac{-3}{x+1}+\dfrac{-4}{y+1}+\dfrac{-5}{z+1} =-11$
$\dfrac{3}{x+1}+\dfrac{4}{y+1}+\dfrac{5}{z+1} =11$

$\dfrac{12}{\frac{3}{x+1}+\frac{4}{y+1}+\frac{5}{z+1}} =\dfrac{12}{11} $ ---(1)

แต่ $\dfrac{3}{x+1}+\dfrac{4}{y+1}+\dfrac{5}{z+1} \leqslant \dfrac{3}{2\sqrt{x}}+\dfrac{4}{2\sqrt{y}}+\dfrac{5}{2\sqrt{z}}$

(1) จัดรูป;

$\dfrac{12}{\frac{1}{\sqrt{x}}+\frac{1}{\sqrt{x}}+\frac{1}{\sqrt{x}}+...+\frac{1}{\sqrt{z}}} \geqslant \dfrac{6}{11} $

GM-HM;

$\sqrt[24]{x^3y^4z^5} \geqslant \dfrac{6}{11} $

$x^3y^4z^5\geqslant (\frac{6}{11} )^{24} = (\frac{36}{121} )^{12} > \frac{1}{11^{12}} $

$\therefore x^3y^4z^5 \le \dfrac{1}{11^{12}}$ เป็นเท็จ

พิสูจน์ไว้แล้วในหน้าก่่อนครับ นำมาแสดงให้ดูซ้ำ

BLACK-Dragon 12 พฤศจิกายน 2011 12:10

อ้างอิง:

ข้อความเดิมเขียนโดยคุณ Thgx0312555 (ข้อความที่ 127101)

แต่ $\dfrac{3}{x+1}+\dfrac{4}{y+1}+\dfrac{5}{z+1} \leqslant \dfrac{3}{2\sqrt{x}}+\dfrac{4}{2\sqrt{y}}+\dfrac{5}{2\sqrt{z}}$


$\dfrac{12}{\frac{1}{\sqrt{x}}+\frac{1}{\sqrt{x}}+\frac{1}{\sqrt{x}}+...+\frac{1}{\sqrt{z}}} \geqslant \dfrac{6}{11} $

ลองดูตรงนี้นะครับ

$\dfrac{1}{\dfrac{3}{x+1}+\dfrac{4}{y+1}+\dfrac{5}{z+1}} \ge \dfrac{1}{\dfrac{3}{2\sqrt{x}}+\dfrac{4}{2\sqrt{y}}+\dfrac{5}{2\sqrt{z}}}$

$\dfrac{6}{11} \ge \dfrac{12}{\dfrac{3}{\sqrt{x}}+\dfrac{4}{\sqrt{y}}+\dfrac{5}{\sqrt{z}}}$

มันคงกลับข้างตรงนี้แหละ :)

จูกัดเหลียง 12 พฤศจิกายน 2011 13:48

อ้างอิง:

ข้อความเดิมเขียนโดยคุณ จูกัดเหลียง (ข้อความที่ 127044)
Let $x,y,z>0$ such that $xy+yz+zx=xyz$
Prove $$\frac{x+2y-z}{z+x}+\frac{y+2z-x}{x+y}+\frac{z+2x-y}{y+z}\ge \frac{xyz}{x+y+z}$$

Let $a=\frac{1}{x},b=\frac{1}{y},c=\frac{1}{z}$ Then $a+b+c=1$
by Cauchy $$\frac{a}{a+b}+\frac{b}{b+c}+\frac{c}{c+a}\ge \frac{(a+b+c)^2}{2(ab+bc+ca)}=\frac{1}{2(ab+bc+ca)}$$
$$\therefore \frac{x}{z+x}+\frac{y}{x+y}+\frac{z}{y+z}\ge \frac{xyz}{2(xy+yz+zx)}$$
and Nesbitt's $$\frac{x}{y+z}+\frac{y}{z+x}+\frac{z}{x+y}\ge \frac{3}{2}$$
$$\Rightarrow \frac{z+x}{y+z}+\frac{y+z}{x+y}+\frac{x+y}{z+x}\ge \frac{xyz}{2(xy+yz+zx)}+\frac{3}{2}$$
$$\frac{x+2y-z}{z+x}+\frac{y+2z-x}{x+y}+\frac{z+2x-y}{y+z}=2\Big(\frac{z+x}{y+z}-\frac{1}{2}+\frac{y+z}{x+y}-\frac{1}{2}+\frac{x+y}{z+x}-\frac{1}{2}\Big)$$ $$ \ge \frac{xyz}{xy+yz+zx}$$

จูกัดเหลียง 12 พฤศจิกายน 2011 13:49

Let $x,y,z>2$ such that $xy+yz+zx=xyz$
Prove $$x+y+z\ge 1+8(x-2)(y-2)(z-2)$$

Thgx0312555 12 พฤศจิกายน 2011 16:44

อ้างอิง:

ข้อความเดิมเขียนโดยคุณ BLACK-Dragon (ข้อความที่ 127102)
ลองดูตรงนี้นะครับ

$\dfrac{1}{\dfrac{3}{x+1}+\dfrac{4}{y+1}+\dfrac{5}{z+1}} \ge \dfrac{1}{\dfrac{3}{2\sqrt{x}}+\dfrac{4}{2\sqrt{y}}+\dfrac{5}{2\sqrt{z}}}$

$\dfrac{6}{11} \ge \dfrac{12}{\dfrac{3}{\sqrt{x}}+\dfrac{4}{\sqrt{y}}+\dfrac{5}{\sqrt{z}}}$

มันคงกลับข้างตรงนี้แหละ :)

ขอบคุณมากครับ

LightLucifer 12 พฤศจิกายน 2011 21:40

#23

BLACK-Dragon 12 พฤศจิกายน 2011 21:58

จริงๆพี่ จูกัดเหลียง เขาเฉลยให้ผมแล้วล่ะครับแต่ยังไม่ได้ลง ตามคุณ LightLucifer

ก็กลายเป็นเราต้องพิสูจน์

$(\dfrac{ab+bc+ca}{abc})(a+b+c) \ge 1+8\dfrac{(a+b-c)(a-b+c)(b-a+c)}{abc}$

$(ab+bc+ca)(a+b+c) \ge abc+8(a+b-c)(a-b+c)(b-a+c)$

แต่ $abc \ge (a+b-c)(a-b+c)(b-a+c)$ (พิสูจน์ได้โดยเปลี่ยน a=p+q,b=q+r,c=r+p)

$(ab+bc+ca)(a+b+c) \ge 9abc \ge abc+8(a+b-c)(a-b+c)(b-a+c)$

AnDroMeDa 12 พฤศจิกายน 2011 22:01

อ้างอิง:

ข้อความเดิมเขียนโดยคุณ จูกัดเหลียง (ข้อความที่ 127104)
Let $x,y,z>2$ such that $xy+yz+zx=xyz$
Prove $$x+y+z\ge 1+8(x-2)(y-2)(z-2)$$

เพิ่งเห็นหุหุ
จาก Hint:ของคุณ LightLucifer ให้ $x=\frac{a+b+c}{a},y=\frac{a+b+c}{b},z=\frac{a+b+c}{c}$
ได้อมการสมมูลกับ $3abc+\sum_{sym}^{}ab^2\geqslant abc+8(a+b-c)(a+c-b)(b+c-a)$
แต่โดยอสมการ A.M.-G.M.ได้ว่า $3abc+\sum_{sym}^{}ab^2\geqslant 9abc$
ซึ่งต้องแสดงว่า $abc\geqslant (a+b-c)(a+c-b)(b+c-a)$
ให้ $a=k+l,b=l+m,c=k+m $ ก็จะได้ว่าเป็นจริงโดยอสมการ A.M.-G.M.
-..-เร็วจริงคุณ Black Dragon

LightLucifer 13 พฤศจิกายน 2011 00:33

เห็นเล่นโจทย์แนวๆนี้กัน
ให้ $x,y,z>0$ ซึ่ง $x+y+z=xyz$
จงพิสูจน์ว่า
$$\frac{1}{x}+\frac{1}{y}+\frac{1}{z} \ge \sqrt{3}$$

AnDroMeDa 13 พฤศจิกายน 2011 01:16

อ้างอิง:

ข้อความเดิมเขียนโดยคุณ LightLucifer (ข้อความที่ 127169)
เห็นเล่นโจทย์แนวๆนี้กัน
ให้ $x,y,z>0$ ซึ่ง $x+y+z=xyz$
จงพิสูจน์ว่า
$$\frac{1}{x}+\frac{1}{y}+\frac{1}{z} \ge \sqrt{3}$$

ให้ $p=x+y+z,q=xy+yz+xz,r=xyz$ โจทย์กำหนด $p=r$ และอสมการเริ่มต้นสมมูลกับ
$q\geqslant \sqrt{3}r $
$\Leftrightarrow q^2\geqslant 3r^2=3pr$
ซึ่ง $q^2\geqslant 3pr \Leftrightarrow (xy+yz+xz)^2\geqslant 3xyz(x+y+z)$
แต่จากอสมการโคชีได้ $(a+b+c)^2\geqslant 3(ab+bc+ac)$ ให้ $a=xy,b=yz,c=xz$ จะได้อสมการที่ต้องการ

Keehlzver 13 พฤศจิกายน 2011 03:40

ลองทำโจทย์ของผมดูครับ ไม่ยากเท่าการทำใจรอให้น้ำลดแน่นอนครับ :dry:

ข้อ 1. นิยามให้ $S_{n}=a^n+b^n+c^n$ โดยที่ $a,b,c > 0$ จงพิสูจน์ว่า $\frac{(S_{5}+S_{4}+S_{2})(S_{4}+S_{2}+S_{1})}{(ab+bc+ca)^3}\geq 3$

ข้อ 2. $x,y,z > 0$ จงพิสูจน์ว่า $(9(x+y+z)+9)(3(xy+yz+zx)+3)\leq (x+y+z+3)^2+8(x+y+z)^3$

ข้อ 3. กำหนดให้ $x,y,z > 0$ และ $xyz=1$ จงพิสูจน์ว่า $\frac{x^5+x^2}{(y+z)^2}+\frac{y^5+y^2}{(z+x)^2}+\frac{z^5+z^2}{(x+y)^2}\geq \frac{3(x^3y+y^3z+z^3x)}{2(x+y+z)}$

ข้อ 4. $a,b,c>0$ จงพิสูจน์ว่า $\frac{(a^3+b^3)(a+b)}{(a^3-b^3)(a-b)}+\frac{(b^3+c^3)(b+c)}{(b^3-c^3)(b-c)}+\frac{(c^3+a^3)(c+a)}{(c^3-a^3)(c-a)} \geq 3+\frac{ab(a+b)^2}{(a-b)^2(2a^2-ab+2b^2)}+\frac{bc(b+c)^2}{(b-c)^2(2b^2-bc+2c^2)}+\frac{ca(c+a)^2}{(c-a)^2(2c^2-ca+2a^2)}$

ข้อ 5. $a,b,c>0$ จงพิสูจน์ว่า $\frac{1}{a^2-ab+b^2}+\frac{1}{b^2-bc+c^2}+\frac{1}{c^2-ca+a^2}+\frac{2}{(a+b)^2+2ab}+\frac{2}{(b+c)^2+2bc}+\frac{2}{(c+a)^2+2ca}\geq \frac{2(a^4-b^4)}{(a^3-b^3)(a^3+b^3)}+\frac{2(b^4-c^4)}{(b^3-c^3)(b^3+c^3)}+\frac{2(c^4-a^4)}{(c^3-a^3)(c^3+a^3)}$

ข้อ 6. $a,b,c>0$ จงพิสูจน์ว่า $\sqrt{2a^4+2a^2b^2+2b^4}+\sqrt{2b^4+2b^2c^2+2c^4}+\sqrt{2c^4+2c^2a^2+2a^4} \geq \sqrt{\frac{ab(a^2+b^2)^2}{(a+b)^2}+\frac{a^5-b^5}{a-b}}+\sqrt{\frac{bc(b^2+c^2)^2}{(b+c)^2}+\frac{b^5-c^5}{b-c}}+\sqrt{\frac{ca(c^2+a^2)^2}{(c+a)^2}+\frac{c^5-a^5}{c-a}}$

(ปล.ผมอยู่ต่างจังหวัด เข้าเว็บลำบากมาก :o)

BLACK-Dragon 13 พฤศจิกายน 2011 10:55

อ้างอิง:

ข้อความเดิมเขียนโดยคุณ Keehlzver (ข้อความที่ 127172)
ข้อ 1. นิยามให้ $S_{n}=a^n+b^n+c^n$ โดยที่ $a,b,c > 0$ จงพิสูจน์ว่า $\dfrac{(S_{5}+S_{4}+S_{2})(S_{4}+S_{2}+S_{1})}{(ab+bc+ca)^3}\geq 3$


AnDroMeDa 13 พฤศจิกายน 2011 12:17

อ้างอิง:

ข้อความเดิมเขียนโดยคุณ Keehlzver (ข้อความที่ 127172)
ลองทำโจทย์ของผมดูครับ ไม่ยากเท่าการทำใจรอให้น้ำลดแน่นอนครับ :dry:

ข้อ 5. $a,b,c>0$ จงพิสูจน์ว่า $\frac{1}{a^2-ab+b^2}+\frac{1}{b^2-bc+c^2}+\frac{1}{c^2-ca+a^2}+\frac{2}{(a+b)^2+2ab}+\frac{2}{(b+c)^2+2bc}+\frac{2}{(c+a)^2+2ca}\geq \frac{2(a^4-b^4)}{(a^3-b^3)(a^3+b^3)}+\frac{2(b^4-c^4)}{(b^3-c^3)(b^3+c^3)}+\frac{2(c^4-a^4)}{(c^3-a^3)(c^3+a^3)}$

(ปล.ผมอยู่ต่างจังหวัด เข้าเว็บลำบากมาก :o)

จับคู่ก่อน จะแสดงว่า $\frac{1}{a^2-ab+b^2}+\frac{2}{(a+b)^2+2ab}\geqslant \frac{2(a^4-b^4)}{(a^3-b^3)(a^3+b^3)} $แต่จาก $(a-b)^2\geqslant 0\Leftrightarrow \frac{2}{(a+b)^2+2ab} \geqslant \frac{1}{a^2+ab+b^2}\Leftrightarrow \frac{1}{a^2-ab+b^2}+\frac{2}{(a+b)^2+2ab} \geqslant \frac{1}{a^2-ab+b^2}+\frac{1}{a^2+ab+b^2}= \frac{2(a^2+c^2)}{(a^2-ac+c^2)(a^2+ac+c^2)}=\frac{2(a^4-c^4)}{(a^3-c^3)(a^3+c^3)} $
ในทำนองเดียวกัน.....จะได้ $\sum_{cyc}(\frac{1}{a^2-ab+b^2}+\frac{2}{(a+b)^2+2ab})\geqslant \sum_{cyc}\frac{2(a^4-b^4)}{(a^3-b^3)(a^3+b^3)} $

AnDroMeDa 13 พฤศจิกายน 2011 12:57

อ้างอิง:

ข้อความเดิมเขียนโดยคุณ Keehlzver (ข้อความที่ 127172)
ลองทำโจทย์ของผมดูครับ ไม่ยากเท่าการทำใจรอให้น้ำลดแน่นอนครับ :dry:
ข้อ 6. $a,b,c>0$ จงพิสูจน์ว่า $\sqrt{2a^4+2a^2b^2+2b^4}+\sqrt{2b^4+2b^2c^2+2c^4}+\sqrt{2c^4+2c^2a^2+2a^4} \geq \sqrt{\frac{ab(a^2+b^2)^2}{(a+b)^2}+\frac{a^5-b^5}{a-b}}+\sqrt{\frac{bc(b^2+c^2)^2}{(b+c)^2}+\frac{b^5-c^5}{b-c}}+\sqrt{\frac{ca(c^2+a^2)^2}{(c+a)^2}+\frac{c^5-a^5}{c-a}}$

(ปล.ผมอยู่ต่างจังหวัด เข้าเว็บลำบากมาก :o)

จะแสดงว่า$\sqrt{2a^4+2a^2b^2+2b^4}\geqslant \sqrt{\frac{ab(a^2+b^2)^2}{(a+b)^2}+\frac{a^5-b^5}{a-b}}$ กระจาย $\Leftrightarrow (a^3-b^3)^2\geqslant 0$
ในทำนองเดียวกันจะได้ $\sum_{cyc}\sqrt{2a^4+2a^2b^2+2b^4}\geqslant \sum_{cyc}\sqrt{\frac{ab(a^2+b^2)^2}{(a+b)^2}+\frac{a^5-b^5}{a-b}}$
ข้อนี้ดูเหมือนเละแต่กระจายออกมาสวย คุณ Keehlzver เอาโจทย์มาจากที่ไหนหรอครับ

BLACK-Dragon 13 พฤศจิกายน 2011 12:59

อ้างอิง:

ข้อความเดิมเขียนโดยคุณ AnDroMeDa (ข้อความที่ 127180)
จะแสดงว่า$\sqrt{2a^4+2a^2b^2+2b^4}\geqslant \sqrt{\frac{ab(a^2+b^2)^2}{(a+b)^2}+\frac{a^5-b^5}{a-b}}$ กระจาย $\Leftrightarrow (a^3-b^3)^2\geqslant 0$
ในทำนองเดียวกันจะได้ $\sum_{cyc}\sqrt{2a^4+2a^2b^2+2b^4}\geqslant \sum_{cyc}\sqrt{\frac{ab(a^2+b^2)^2}{(a+b)^2}+\frac{a^5-b^5}{a-b}}$
ข้อนี้ดูเหมือนเละแต่กระจายออกมาสวย คุณ Keehlzver เอาโจทย์มาจากที่ไหนหรอครับ

ไวจริงครับ กำลังจะโำพสต์เลย :great:

BLACK-Dragon 13 พฤศจิกายน 2011 13:56

อ้างอิง:

ข้อความเดิมเขียนโดยคุณ Keehlzver (ข้อความที่ 127172)

ข้อ 4. $a,b,c>0$ จงพิสูจน์ว่า $\dfrac{(a^3+b^3)(a+b)}{(a^3-b^3)(a-b)}+\dfrac{(b^3+c^3)(b+c)}{(b^3-c^3)(b-c)}+\dfrac{(c^3+a^3)(c+a)}{(c^3-a^3)(c-a)} \geq 3+\dfrac{ab(a+b)^2}{(a-b)^2(2a^2-ab+2b^2)}+\dfrac{bc(b+c)^2}{(b-c)^2(2b^2-bc+2c^2)}+\dfrac{ca(c+a)^2}{(c-a)^2(2c^2-ca+2a^2)}$

ผมพิสูจน์ได้แค่อันนี้อ่ะครับ

$$\dfrac{(a^3+b^3)(a+b)}{(a^3-b^3)(a-b)}+\dfrac{(b^3+c^3)(b+c)}{(b^3-c^3)(b-c)}+\dfrac{(c^3+a^3)(c+a)}{(c^3-a^3)(c-a)} \geq \dfrac{ab(a+b)^2}{(a-b)^2(2a^2-ab+2b^2)}+\dfrac{bc(b+c)^2}{(b-c)^2(2b^2-bc+2c^2)}+\dfrac{ca(c+a)^2}{(c-a)^2(2c^2-ca+2a^2)}$$

$$\displaystyle \sum_{cyc} \dfrac{(a+b)^2(a^2-ab+b^2)}{(a-b)^2(a^2+ab+b^2)} \geq\sum_{cyc} \dfrac{(a+b)^2}{3(a-b)^2} \geq \sum_{cyc} \dfrac{ab(a+b)^2}{(a-b)^2(2a^2-ab+2b^2)}$$


AnDroMeDa 13 พฤศจิกายน 2011 14:37

อ้างอิง:

ข้อความเดิมเขียนโดยคุณ Keehlzver (ข้อความที่ 127172)

ข้อ 4. $a,b,c>0$ จงพิสูจน์ว่า $\frac{(a^3+b^3)(a+b)}{(a^3-b^3)(a-b)}+\frac{(b^3+c^3)(b+c)}{(b^3-c^3)(b-c)}+\frac{(c^3+a^3)(c+a)}{(c^3-a^3)(c-a)} \geq 3+\frac{ab(a+b)^2}{(a-b)^2(2a^2-ab+2b^2)}+\frac{bc(b+c)^2}{(b-c)^2(2b^2-bc+2c^2)}+\frac{ca(c+a)^2}{(c-a)^2(2c^2-ca+2a^2)}$

จะแสดงว่า$\frac{(a^3+b^3)(a+b)}{(a^3-b^3)(a-b)}\geqslant 1+\frac{ab(a+b)^2}{(a-b)^2(2a^2-ab+2b^2)}$
กระจาย$\Leftrightarrow 3a^5b-5a^4b^2+4a^3b^3-5a^2b^4+3ab^5=ab(3(a-b)^4+7ab(a-b)^2)\geqslant 0 $ Obvious,it's true.
ดังนั้น $\sum_{cyc}\frac{(a^3+b^3)(a+b)}{(a^3-b^3)(a-b)}\geqslant 3+\sum_{cyc}\frac{ab(a+b)^2}{(a-b)^2(2a^2-ab+2b^2)}$

จูกัดเหลียง 13 พฤศจิกายน 2011 17:35

2. โจทย์ผิดป่าวครับ

จูกัดเหลียง 13 พฤศจิกายน 2011 19:29

อ้างอิง:

ข้อความเดิมเขียนโดยคุณ Keehlzver (ข้อความที่ 127172)

ข้อ 3. กำหนดให้ $x,y,z > 0$ และ $xyz=1$ จงพิสูจน์ว่า $$\frac{x^5+x^2}{(y+z)^2}+\frac{y^5+y^2}{(z+x)^2}+\frac{z^5+z^2}{(x+y)^2}\geq \frac{3(x^3y+y^3z+z^3x)}{2(x+y+z)}$$


Keehlzver 13 พฤศจิกายน 2011 21:59

ทุกคนมีการพัฒนาไปมากครับ ยินดีด้วยครับ :great:

โจทย์ข้อ 2. ไม่ผิดครับ ง่ายสุดเลยด้วยครับ แต่ไม่ยักมีคนทำ :D

อสมการในรูป $(a_{1}b_{1}+a_{2}b_{2}+a_{3}b_{3}) \geq \frac{1}{3}(a_{1}+a_{2}+a_{3})(b_{1}+b_{2}+b_{3})$ จะใช้ได้ก็ต่อเมื่อ $a_{1}\geq a_{2}\geq a_{3}$ และ $b_{1}\geq b_{2} \geq b_{3}$ นะครับ
http://en.wikipedia.org/wiki/Chebysh...sum_inequality

นั่นหมายความว่า อย่าลืมพิสูจน์ $(\frac{x}{y+z})^2 \geq (\frac{y}{z+x})^2 \geq (\frac{z}{x+y})^2$ นะครับ

และก็อีกอย่างคือ มีที่ผิดตรงวิธีทำตรงข้อ 2
อสมการนี้ $\frac{1}{3}\Big(\Big(\frac{x}{y+z}\Big)^2+\Big(\frac{y}{z+x}\Big)^2+\Big(\frac{z}{x+y}\Big)^2\Big)(x^3+y^3+z^3+3) \geq \frac{3}{4}(x^2+y^2+z^2+x+y+z)$ มันไม่จริง $(a,b,c)=(1,1,1)$ ก็ขัดแย้งแล้วครับ :)

(ปล.โจทย์ทุกข้อผมแต่งเองครับ)

BLACK-Dragon 13 พฤศจิกายน 2011 22:21

อ้างอิง:

ข้อความเดิมเขียนโดยคุณ Keehlzver (ข้อความที่ 127212)
ทุกคนมีการพัฒนาไปมากครับ ยินดีด้วยครับ :great:

โจทย์ข้อ 2. ไม่ผิดครับ ง่ายสุดเลยด้วยครับ แต่ไม่ยักมีคนทำ :D

อสมการในรูป $(a_{1}b_{1}+a_{2}b_{2}+a_{3}b_{3}) \geq \frac{1}{3}(a_{1}+a_{2}+a_{3})(b_{1}+b_{2}+b_{3})$ จะใช้ได้ก็ต่อเมื่อ $a_{1}\geq a_{2}\geq a_{3}$ และ $b_{1}\geq b_{2} \geq b_{3}$ นะครับ
http://en.wikipedia.org/wiki/Chebysh...sum_inequality

นั่นหมายความว่า อย่าลืมพิสูจน์ $(\frac{x}{y+z})^2 \geq (\frac{y}{z+x})^2 \geq (\frac{z}{x+y})^2$ นะครับ

และก็อีกอย่างคือ มีที่ผิดตรงวิธีทำตรงข้อ 2
อสมการนี้ $\frac{1}{3}\Big(\Big(\frac{x}{y+z}\Big)^2+\Big(\frac{y}{z+x}\Big)^2+\Big(\frac{z}{x+y}\Big)^2\Big)(x^3+y^3+z^3+3) \geq \frac{3}{4}(x^2+y^2+z^2+x+y+z)$ มันไม่จริง $(a,b,c)=(1,1,1)$ ก็ขัดแย้งแล้วครับ :)

(ปล.โจทย์ทุกข้อผมแต่งเองครับ)

คาราวะ 10 จอกเลยครับ :great:

ขอนอกเรื่องนิดนึงนะครับ น้ำที่บ้านเริ่มลดหรือยังครับคุณ Keehlzver


เวลาที่แสดงทั้งหมด เป็นเวลาที่ประเทศไทย (GMT +7) ขณะนี้เป็นเวลา 02:38

Powered by vBulletin® Copyright ©2000 - 2024, Jelsoft Enterprises Ltd.
Modified by Jetsada Karnpracha